Đến nội dung

mathandyou

mathandyou

Đăng ký: 30-05-2013
Offline Đăng nhập: 10-02-2018 - 23:31
****-

#483528 Bài viết về đường tròn nội tiếp tam giác.

Gửi bởi mathandyou trong 16-02-2014 - 20:17

Xin chào các thành viên VMF.
Đây là bài viết của mình về những tính chất đường tròn nội tiếp.Đó hầu hết là những bài tập mình đã làm,phần nhiều là thầy dạy.Mình trình bày lại để thể hiện những hướng giải khi đứng trước các bài toán hình học của mình,chia sẻ với mọi người để trao đổi kinh nghiệm,mình chú trọng thể hiện phương pháp.Các bài toán thuộc diện "cũ" nhưng mình đem ra để mổ xẻ lại mong phần nào giúp mọi người hiểu thêm về các vấn đề đó.Bài viết còn khá thô sơ cũng như kiến thức hạn hẹp nên không tránh khỏi sai sót.Rất mong được sự đóng góp ý kiến từ mọi người.Đây là lần đầu mình làm một bài viết về hình học,vẽ hình còn xấu,cách trình bày chưa đẹp và cân đối,mình sử dụng Mathtype vì chưa có thời gian nghiên cứu nhiều về latex để trình bày hình học...Mong mọi người giúp đỡ.Chân thành cám ơn! :)
Rất tiếc là những bài toán hay như IMO 2011,2013 mình chưa đưa vào.Các tính chất về các đường đẳng giác,hàng điểm điều hòa,cực và đối cực mình cũng chưa có thời gian để viết tiếp vì sắp tới kiểm tra bài trên lớp nhiều.Hẹn gặp các bạn ở dịp khác :)

 

File gửi kèm




#482358 Chứng minh :BP=CQ.

Gửi bởi mathandyou trong 10-02-2014 - 12:24

Bài này cũng dễ thôi tại lâu ngày không post bài nên post kẻo quên.:)

Bổ đề của chú cm đơn giản là qua I kẻ đường thẳng song song với BC cắt AC,AB tại P và Q.Dựa vào 2 tứ giác nội tiếp ta chứng minh hai góc IPO và IQO bằng nhau nên được PI=QI.Suy ra I thuộc AM.

Đây là shortlish IMO 2005 thì phải.




#482300 Chứng minh :BP=CQ.

Gửi bởi mathandyou trong 09-02-2014 - 20:54

Đường trung tuyến AM của tam giác ABC cắt đường tròn nội tiếp (I) của tam giác tại K và L.Từ K,L kẻ đường thẳng song song với BC cắt (I) tại X,Y.AX và AY cắt BC tại P và Q.Chứng minh :BP=CQ.

 

 




#480069 Giải phương trình: $\sqrt[3]{x^{2}-1}+x=\s...

Gửi bởi mathandyou trong 30-01-2014 - 17:37

PT $< = > (\sqrt{x^3-2}-5)-(\sqrt[3]{x^2-1}-2)-(x-3)=0< = > \frac{(x^3-27)}{\sqrt{x^3-2}+5}-\frac{x^2-9}{\sqrt[3]{(x^2-1)^2+2\sqrt[3]{x^2-1}+4}}-(x-3)=0< = > (x-3)(\frac{x^2+3x+9}{\sqrt{x^3-2}+5}-\frac{x+3}{\sqrt[3]{(x^2-1)^2}+\sqrt[3]{2(x^2-1}+4}-1)=0< = > x=3$

Sao bạn chứng minh đoạn tô đỏ vô nghiệm?




#479875 giải hệ

Gửi bởi mathandyou trong 29-01-2014 - 17:02

Em đã sửa, còn cái cách bần cùng sinh dâm tặc đó thôi àk @@. Mà nếu làm theo kiểu hàm thì có cách nào chữa không anh :))

Có cách là:Chú cần chứng minh cả x và t đều lớn hơn 0.Khi đó xét hàm trên khoảng từ 0 đến dương vô cùng.




#479867 giải hệ

Gửi bởi mathandyou trong 29-01-2014 - 16:12

Huy giải chưa đúng rồi.Hàm này tuy đồng biến nhưng TXĐ ở hai vế là khác nhau và VP bị gián đoạn tại điểm x=0.




#479825 CĐT Olympic toán 10 THPT Chuyên Lê Hồng Phong TpHCM 2014 (Lần 2)

Gửi bởi mathandyou trong 29-01-2014 - 12:58

Bổ đề. Nếu $p|a^n-b^n$ với $a,b,n,p \in \mathbb{N}^*$ và $p$ nguyên tố. Gọi $k$ là số nguyên dương nhỏ nhất thoả mãn $p|a^k-b^k$. Khi đó $k|n$.

Chứng minh. Đặt $n=kq+r$ với $q,r \in \mathbb{N},0 \le r \le k-1$. Ta có $$a^n-b^n=a^{kq+r}-b^{kq+r}= a^r \left( a^{kq}-b^{kq} \right)+ b^{kq} \left( b^r-a^r \right).$$

Ta suy ra $p|b^r-a^r$. Theo điều kiện nhỏ nhất của $k$ thì ta dẫn đến $r=0$. Như vậy $k|n$.

Lời giải. Nhận thấy $n=1$ thoả mãn bài toán.

Nếu $n \ge 2$ thì ta gọi $p$ là ước nguyên tố nhỏ nhất của $n$, khi đó $p|n$ nên $p|3^n-2^n$.

Gọi $k$ là số nguyên dương nhỏ nhất thoả mãn $p|3^k-2^k$. Theo bổ đề thì $k|n$.

Dễ thấy rằng $3 \nmid n,2 \nmid n$ nên theo định lý Fermat nhỏ $p|3^{p-1}-2^{p-1}$ nên $k|p-1$ suy ra $p>k$.

Ta sẽ chứng minh rằng $\gcd (n,k)=1$.

Giả sử ngược lại, $r| \gcd (n,k)$ với $r$ nguyên tố, khi đó $r|n$ và $r<k<p$, điều này mâu thuẫn với điều kiện nhỏ nhất của $p$.

Do đó $\gcd (n,k)=1$. Mà $k|n$ nên $k=1$. Khi đó $p|3-2$ hay $p|1$, mâu thuẫn vì $p$ nguyên tố.

Vậy $n=1$. $\blacksquare$

Ta còn một cách như này:

Gọi p là ước nguyên tố nhỏ nhất của n.Khi đó:$p\geq 5$

Gọi $a \in z+$ sao cho:$2a \equiv 1 (mod p)$

Vì:$3^n \equiv 2^n (mod p)$ nên $ (3a)^n \equiv 1 (mod )$.

Theo cụ Fermat thì:$(3a)^{p-1} \equiv 1 (mod p)$

Gọi $h$ là cấp của $3a$ (mod p) thì:$n \vdots h$ và $p-1 \vdots h$ và $p-1 \vdots h$

Từ đó $h=1$ nên $3a \equiv 1 (mod p)$.

Do đó $a \vdots p$.Vô lí




#479792 Các điểm đặc biệt trong tam giác

Gửi bởi mathandyou trong 29-01-2014 - 10:04

Hôm qua khi làm bài về một bổ đề thì mình phát hiện ra một tính chất thế này.

Cho tam giác nhọn ABC.$G_e,N_a$ lần lượt là điểm Gergonne và điểm Nagel của tam giác.$O,H,G$ lần lượt là tâm đường tròn ngoại tiếp,trực tâm và trọng tâm của tam giác.$E$ là trung điểm $OH$.Chứng minh:trung điểm $GE$ nằm trên $G_eN_a$.

 




#479606 Tìm Max:$P = 6( y+z-x)+27xyz$

Gửi bởi mathandyou trong 28-01-2014 - 11:42

Èo...chú đừng chém,Bđt như chú mà làm ko được mới ghê.Chị Nam hỏi a đấy,A kêu a ko biết mà nó cứ lèo nhèo nên a lên đây hỏi cho nó




#479568 Tìm Max:$P = 6( y+z-x)+27xyz$

Gửi bởi mathandyou trong 28-01-2014 - 09:44

Cho ba số thực không âm $x,y,z$ thỏa:$x^{2}+y^{2}+z^{2}=1$
Tìm giá trị lớn nhất của $P = 6( y+z-x)+27xyz$

 




#479450 $\left\{\begin{matrix}x+\sqrt{x^...

Gửi bởi mathandyou trong 27-01-2014 - 19:44



Lời giải :

Xét $x<0$, ta luôn có $y+\sqrt{y^2+3}>0$ nên $x+\sqrt{x^2-3}>0\Rightarrow x>-\sqrt{x^2-3}\Rightarrow x^2<x^2-3$, điều này mâu thuẫn. Như vậy phải có $x>0$.

Đặt $\sqrt{x^2-3}=a\geq 0\Rightarrow x^{2}=a^2+3\Rightarrow x=\pm \sqrt{a^2+3}$. Nhưng vì $x>0$ nên $x=\sqrt{a^2+3}$

Khi đó, phương trình đầu trở thành $a+\sqrt{a^2+3}=y+\sqrt{y^2+3}$. Xét hàm số $f(t)=t+\sqrt{t^2+3}$, kiểm tra được hàm này đồng biến trên $\mathbb{R}$ nên $f(a)=f(y)\Leftrightarrow a=y\Leftrightarrow x^2-y^2=3$.

Ta được hệ $\left\{\begin{matrix} x^3-y^3=3(x-y)+4\\ x^2-y^2=3 \end{matrix}\right.$

Đặt $u=x+y,v=x-y$ thì $xy=\dfrac{u^2-v^2}{4}$

Do đó $\left\{\begin{matrix} x^3-y^3=3(x-y)+4\\ x^2-y^2=3 \end{matrix}\right.\Leftrightarrow \left\{\begin{matrix} (x-y)\left [ (x+y)^2-xy-3 \right ]=4\\ (x-y)(x+y)=3 \end{matrix}\right.\Leftrightarrow \left\{\begin{matrix} v\left ( u^2-\dfrac{u^2-v^2}{4}-3 \right )=4\\ uv=3 \end{matrix}\right.\Leftrightarrow \left\{\begin{matrix} v(3u^2+v^2-12)=16\\ uv=3 \end{matrix}\right.\Leftrightarrow \left\{\begin{matrix} 9u=-v^3+12v+16\\ uv=3 \end{matrix}\right.\Leftrightarrow \left\{\begin{matrix} uv=3\\ v^4-12v^2-16v+27=0 \end{matrix}\right.\Leftrightarrow (u,v)=(3,1)\Leftrightarrow (x,y)=(2,1)$

 

Hàm $f(t)$ chỉ đồng biến trên R+ thôi Huy và hàm đồng biến+liên tục thì mới có kq đó.nhưng cơ bản như vậy là được




#479432 Chứng minh $E,F,Y,Z$ đồng viên.

Gửi bởi mathandyou trong 27-01-2014 - 17:45

Bài này không cần Menelaus như vậy:

Ta có:$(KDBC)=-1.$

Gọi $K'$ là giao điểm của $EF$ và $BC$.Vì $AD,BE,CF$ đồng qui nên:$(K'DBC)=-1$.

Từ hai điều này ta suy ra $K' \equiv K$ nên có ngay đpcm.




#479417 Tài liệu về Hàng điểm điều hòa và phép nghịch đảo.

Gửi bởi mathandyou trong 27-01-2014 - 16:45

Đây là một số file về hàng điểm điều hòa mình có được.

File gửi kèm




#479206 $p^{p+1}+(p+1)^{p}$ không là số chính phương.

Gửi bởi mathandyou trong 26-01-2014 - 19:31

Mình có hướng giải thế này:

Đặt $p^{p+1}+(p+1)^p=a^2$

Ta đễ có:$a^2-1 \vdots p$ và $a^2-1 \vdots p+1$

Do đó:$a=kp(p+1)$

*Nếu $p \equiv 1 (mod4)$ thì $a^2 \equiv 3 (mod 4)$ (vô lí)

*Nếu $p \equiv 3 (mod4)$

Khi đó:$kp^2+kp+1-a^2=0$,ta chỉ cần chứng minh pt này vô nghiệm là được.Mình có hướng thế này:

Xét $\delta=k^2-4k(1-a^2)$

pt có nghiệm nguyên khi và chỉ khi:$k^2-4k(1-a^2)=m^2$

hay:$[k-2(1-a^2)]^2=m^2+4(1-a^2)^2$

Đến đây bạn dùng nghiệm của pt pitago rồi thế vào cm điều vô lí.




#479140 Chứng minh rằng $AD,BE,CF$ đồng quy.

Gửi bởi mathandyou trong 26-01-2014 - 12:31

Đây là định lí Kariya.@@

Qua mathlinks hỏi thì người ta nói vậy,mở cuốn sách hình của MS ra thì thấy nó.

Ngoài ra không biết dùng desargue có chứng minh được không??